Problem Rankings

Rank Source Description Elo Rating
3581 AIME 2017 II Q5 A set contains four numbers. The six pairwise sums... 1483.26
3582 AIME 2001 I Q5 An equilateral triangle is inscribed in the ellips... 1483.26
3583 AMC 8 2013 A Q2 A sign at the fish market says, '50\% off, today o... 1483.24
3584 AMC 8 1996 A Q22 The horizontal and vertical distances between adja... 1483.23
3585 AMC 8 1991 A Q4 If \(991+993+995+997+999=5000-N\), then \(N=\) ... 1483.23
3586 AMC 12 1982 A Q5 Two positive numbers \(x\) and \(y\) are in the ra... 1482.75
3587 AMC 10 2009 B Q9 Segment \( BD\) and \( AE\) intersect at \( C\), a... 1482.73
3588 AMC 8 1989 Q If \(a = - 2\), the largest number in the set \(- ... 1482.73
3589 AMC 8 2008 A Q5 Barney Schwinn notices that the odometer on his bi... 1482.73
3590 AMC 8 1998 A Q3 \( \cfrac{\cfrac{3}{8}+\cfrac{7}{8}}{\cfrac{4}{5}}... 1482.72
3591 AMC 8 2011 A Q10 The taxi fare in Gotham City is \(\$2.40\) for the... 1482.71
3592 AMC 10 2003 A Q2 Members of the Rockham Soccer League buy socks and... 1482.60
3593 AMC 10 2010 B Q6 A circle is centered at \( O\), \( \overline{AB}\)... 1482.57
3594 AMC 8 1992 A Q2 Which of the following is not equal to \(\dfrac{5}... 1482.56
3595 AMC 12 2023 B Q3 A \(3-4-5\) right triangle is inscribed in circle ... 1482.56
3596 AMC 8 2001 A Q5 On a dark and stormy night Snoopy suddenly saw a f... 1482.54
3597 IMC 2005 Q6 A shop advertises 'Buy one, get one at half price'... 1482.47
3598 AMC 8 2008 A Q10 The average age of the \(6\) people in Room A is \... 1482.08
3599 AMC 10 2012 B Q15 In a round-robin tournament with \(6\) teams, each... 1482.04
3600 AMC 8 1990 A Q9 The grading scale shown is used at Jones Junior Hi... 1481.98